Matura rozszerzona z matematyki (maj 2023) – rozwiązania

Poniżej prezentujemy rozwiązania wszystkich zadań maturalnych z matematyki dla poziomu rozszerzonego, z którymi abiturienci mierzyli się 12. maja 2023 r. Zadania pochodzą z arkusza opublikowanego w serwisie Centralnej Komisji Egzaminacyjnej.

Zadanie 1. (0-1)
Granica \lim\limits_{x\to 1}\dfrac{x^3-1}{(x-1)(x+2)} jest równa
   A. (-1)
   B. 0
   C. \frac{1}{3}
   D. 1


Rozkładamy licznik na czynniki (różnica sześcianów) i skracamy:

(1)   \begin{eqnarray*} \lim\limits_{x\to 1}\dfrac{x^3-1}{(x-1)(x+2)}&=&\lim\limits_{x\to 1}\dfrac{(x-1)(x^2+x+1)}{(x-1)(x+2)}=\nonumber\\ &=&\lim\limits_{x\to 1}\dfrac{x^2+x+1}{x+2}=\dfrac{1+1+1}{1+2}=1.\nonumber \end{eqnarray*}

Zatem poprawną jest odpowiedź D.



Zadanie 2. (0-1)
Dane są wektory \vec{u} =[4,-5] oraz \vec{v} =[-1,-5]. Długość wektora \vec{u}-4\vec{v} jest równa
   A. 7
   B. 15
   C. 17
   D. 23


Obliczamy współrzędne interesującego nas wektora:

    \[\vec{u}-4\vec{v}=[4,-5]-4\cdot[-1,-5]=[4-(-4),-5-(-20)]=[8,15].\]

Długość tego wektora wynosi \sqrt{8^2+15^2}=\sqrt{64+225}=\sqrt{289}=17.

Poprawną odpowiedzią jest C.



Zadanie 3. (0-1)
Punkty A, B, C, D, E leżą na okręgu o środku S. Miara kąta BCD jest równa 110^\circ, a miara kąta BDA jest równa 35^\circ (zobacz rysunek).

\begin{tikzpicture}[scale=1.3] \draw (0,0) circle (2cm); \coordinate (A) at (275:2); \coordinate (B) at (345:2); \coordinate (C) at (55:2); \coordinate (D) at (125:2); \coordinate (E) at (200:2);  \foreach \x/\w in {A/275,B/345,C/55,D/125,E/200}   \draw[fill=black] (\x) circle (1pt) node at (\w:2.28) {$\x$};  \draw[fill=black] (0,0) circle (1pt) node[below right] {$S$}; \draw[thick] (A)--(B)--(C)--(D)--(E)--cycle; \draw (B)--(D)--(A);  \draw[thin,shift={(C)}] (180:0.7) arc (180:290:0.7) node[scale=0.75] at (235:0.4) {$110^\circ$}; \draw[thin,shift={(D)}] (290:1.3) arc (290:325:1.3) node[scale=0.8] at (307:1) {$35^\circ$}; \draw[thin,shift={(E)}] (327:0.6) arc (327:432:0.6); \end{tikzpicture}

Wtedy kąt DEA ma miarę równą
   A. 100^\circ
   B. 105^\circ
   C. 110^\circ
   D. 115^\circ


\begin{tikzpicture}[scale=1.3] \draw (0,0) circle (2cm); \coordinate (A) at (275:2); \coordinate (B) at (345:2); \coordinate (C) at (55:2); \coordinate (D) at (125:2); \coordinate (E) at (200:2);  \foreach \x/\w in {A/275,B/345,C/55,D/125,E/200}   \draw[fill=black] (\x) circle (1pt) node at (\w:2.28) {$\x$};  \draw[fill=black] (0,0) circle (1pt) node[left] {$S$}; \draw[thick] (A)--(B)--(C)--(D)--(E)--cycle; \draw (B)--(D)--(A);  \draw[thin,shift={(C)}] (180:0.7) arc (180:290:0.7) node[scale=0.75] at (235:0.4) {$110^\circ$}; \draw[thin,shift={(D)}] (290:1.3) arc (290:325:1.3) node[scale=0.8] at (307:1) {$35^\circ$}; \draw[thin,shift={(E)}] (327:0.6) arc (327:432:0.6); \draw[blue] (A)--(0,0)--(B) (B)--(0,0)--(D); \draw[thin,shift={(E)}] (327:0.6) arc (327:432:0.6); \draw[thin,red] (275:0.5) arc (275:345:0.5) node[red,scale=0.8] at (310:0.35) {$70^\circ$}; \draw[thin,red] (345:0.4) arc (345:485:0.4) node[red,scale=0.7] at (55:0.23) {$140^\circ$}; \end{tikzpicture}

Wyznaczamy kąty środkowe: \angle ASB=2\angle ADB=70^\circ. Podobnie (kąt środkowy wklęsły) \angle BSD=2\angle BCD=220^\circ, czyli kąt wypukły \angle BSD=360^\circ-220^\circ=140^\circ. To oznacza, że \angle AED jest połową kąta wklęsłego \angle ASD=210^\circ i właściwą odpowiedzią jest B.



Zadanie 4. (0-1)
Dany jest zbiór trzynastu liczb \{1,2,3,4,5,6,7,8,9,10,11,12,13\}, z którego losujemy jednocześnie dwie liczby. Wszystkich różnych sposobów wylosowania z tego zbioru dwóch liczb, których iloczyn jest liczbą parzystą, jest
   A. \binom{7}{2}+49
   B. \binom{6}{1}\cdot\binom{7}{1}+49
   C. \binom{13}{2}-\binom{7}{2}
   D. \binom{13}{2}-\binom{6}{2}


Losujemy obie liczby jednocześnie, więc ich kolejność nie ma znaczenia przy zliczaniu. Wszystkich możliwych par liczb utworzonych z elementów danego zbioru jest \binom{13}{2}. Odrzucamy z nich te, których iloczyn jest liczbą nieparzystą, czyli gdy obie liczby są nieparzyste; takich zaś par jest dokładnie \binom{7}{2}, bo liczb nieparzystych w danym zbiorze jest siedem.

Odpowiedź C.



Zadanie 5. (0-2)
Wielomian W(x)=7x^3-9x^2+9x-2 ma dokładnie jeden pierwiastek rzeczywisty. Oblicz ten pierwiastek.

W poniższe kratki wpisz kolejno – od lewej do prawej – pierwszą, drugą oraz trzecią cyfrę po przecinku nieskończonego rozwinięcia dziesiętnego otrzymanego wyniku.

\begin{tikzpicture}[yscale=1.5,scale=0.8] \draw[very thick] (0,0) grid (3,1); \end{tikzpicture}


Zauważmy, że W(0)=-2<0 oraz W(1)=5>0, zatem jedyny pierwiastek rzeczywisty danego wielomianu leży w przedziale (0,1). Z twierdzenia o postaci pierwiastków wymiernych wielomianu mającego współczynniki całkowite wynika, że warto sprawdzić wartości postaci

    \[\frac{\text{dzielniki wyrazu wolnego}}{\text{dzielniki wsp. przy najwyższej potędze }x}.\]

Do przedziału (0,1) trafiają tylko dwa takie ułamki: \dfrac{1}{7} oraz \dfrac{2}{7}. Sprawdzamy, że W(\frac{1}{7})\neq 0, zaś W(\frac{2}{7})=0, a ponieważ \dfrac{2}{7}=0,2857\ldots, to do kratek należy wpisać kolejno cyfry 2, 8 i 5.



Zadanie 6. (0-3)
Liczby rzeczywiste x oraz y spełniają jednocześnie równanie x+y=4 i nierówność x^3-x^2y\leqslant xy^2-y^3. Wykaż, że x = 2 oraz y = 2.


Zauważmy, że dla dowolnych liczb rzeczywistych prawdziwa jest nierówność

    \[4(x-y)^2\geqslant 0,\]

przy czym równość zachodzi w niej wyłącznie wtedy, kiedy x=y.

Załóżmy teraz, że liczby x i y spełniają podane w zadaniu warunki. Rozpisując lewą stronę powyższego wyrażenia otrzymamy

    \[4(x^2-xy+y^2-xy)=4(x^2-xy+y^2)-4xy\geqslant 0.\]

Zastępując teraz (dwukrotnie) czynniki 4 sumą x+y dostaniemy

    \[(x+y)(x^2-xy+y^2)\geqslant xy(x+y),\]

czyli x^3+y^3\geqslant x^2y+xy^2 lub inaczej

    \[x^3-x^2y\geqslant xy^2-y^3.\]

Jest to nierówność, której znak jest skierowany przeciwnie niż w przyjętych założeniach z zadania. Jest to możliwe wyłącznie wtedy, gdy obie te nierówności stają się równościami, czyli (zgodnie z uwagą w pierwszym akapicie) dla x=y. A skoro także x+y=4, to musi być x=y=2, co kończy dowód.



Zadanie 7. (0-3)
Dany jest trójkąt prostokątny ABC, w którym |\angle ABC|=90^\circ oraz |\angle CAB|=60^\circ. Punkty K i L leżą na bokach – odpowiednio – AB i BC tak, że |BK|=|BL|=1 (zobacz rysunek). Odcinek KL przecina wysokość BD tego trójkąta w punkcie N, a ponadto |AD|=2.

\begin{tikzpicture}[scale=1.3] \coordinate (A) at (0,4); \coordinate (B) at (0,0); \coordinate (C) at (6.9282,0); \coordinate (K) at (0,1); \coordinate (L) at (1,0); \coordinate (D) at ($(A)!(B)!(C)$); \coordinate (N) at (intersection of B--D and K--L);  \foreach \x/\r in {A/{above},B/{below left},C/{below},D/{above right},K/{left},L/{below},N/{right}}   \draw[fill=black] (\x) circle (0.8pt) node[\r] {$\x$}; \draw (B)--(D); \draw[thin] (A)--(D) node[midway,above right] {$2$}; \draw[thin] (B)--(L) node[midway,below] {$1$}; \draw[thin] (B)--(K) node[midway,left] {$1$}; \draw[thick] (C)--(B)--(A)--cycle; \draw (K)--(L); \draw[thin,shift={(A)}] (270:0.7) arc (270:330:0.7) node at (300:0.5) {$60^\circ$}; \draw[thin,shift={(D)}] (150:0.5) arc (150:240:0.5) node[scale=2] at (195:0.3) {$\cdot$}; \end{tikzpicture}

Wykaż, że |ND|=\sqrt{3}+1.


Wysokość BD w trójkącie ,,ekierkowym” ABC ma długość równą

    \[|BD|=2\cdot\text{tg}\,60^\circ=2\sqrt{3}.\]

\begin{tikzpicture}[scale=1.3] \coordinate (A) at (0,4); \coordinate (B) at (0,0); \coordinate (C) at (6.9282,0); \coordinate (K) at (0,1); \coordinate (L) at (1,0); \coordinate (D) at ($(A)!(B)!(C)$); \coordinate (N) at (intersection of B--D and K--L);  \foreach \x/\r in {A/{above},B/{below left},C/{below},D/{above right},K/{left},L/{below},N/{right}}   \draw[fill=black] (\x) circle (0.8pt) node[\r] {$\x$}; \draw (B)--(D); \draw[thin] (A)--(D) node[midway,above right] {$2$};  \draw[thin] (B)--(K) node[midway,left] {$1$}; \draw[thick] (C)--(B)--(A)--cycle; \draw (K)--(L); \draw[thin,shift={(A)}] (270:0.7) arc (270:330:0.7) node at (300:0.5) {$60^\circ$}; \draw[thin,shift={(D)}] (150:0.5) arc (150:240:0.5) node[scale=2] at (195:0.3) {$\cdot$}; \coordinate (P) at ($(B)!(N)!(L)$); \draw[fill=blue!20] (B)--(N)--(P)--cycle; \draw[red] (N)--(P) node[below] {$P$}; \draw[thin,red] (N)--(P) node[right,midway,scale=0.7] {$x$}; \draw[thin,red] (L)--(P) node[above,midway,scale=0.7] {$x$}; \end{tikzpicture}

Niech P będzie rzutem prostokątnym punktu N na bok BC i rozważmy trójkąty PLN (prostokątny i równoramienny) oraz NBP (kolejny trójkąt ,,ekierkowy”). Jeżeli oznaczymy |PN|=|LP|=x, to |BP|=1-x i wtedy

    \[\frac{x}{1-x}=\frac{|PN|}{|BP|}=\text{tg}\,60^\circ=\sqrt{3},\]

czyli \sqrt{3}(1-x)=x i dalej x(1+\sqrt{3})=\sqrt{3}, a stąd x=\dfrac{\sqrt{3}}{1+\sqrt{3}}. Wtedy \dfrac{|PN|}{|BN|}=\sin 60^\circ=\dfrac{\sqrt{3}}{2}. To oznacza, że

    \[|BN|=\frac{2x}{\sqrt{3}}=\frac{2}{1+\sqrt{3}}=\sqrt{3}-1.\]

Ostatecznie |ND|=|BD|-|BN|=2\sqrt{3}-(\sqrt{3}-1)=\sqrt{3}+1. To kończy dowód.



Zadanie 8. (0-3)
W pojemniku jest siedem kul: pięć kul białych i dwie kule czarne. Z tego pojemnika losujemy jednocześnie dwie kule bez zwracania. Następnie – z kul pozostałych w pojemniku – losujemy jeszcze jedną kulę. Oblicz prawdopodobieństwo wylosowania kuli czarnej w drugim losowaniu.


Schemat rozwiązania najlepiej pokazać w formie grafu. Po pierwszym losowaniu w pojemniku zostanie pięć kul, przy czym mogły to być: 3 kule białe i dwie czarne (3b+2c) albo 4 białe i jedna czarna (4b+1c) albo same kule białe (5b).

\begin{tikzpicture}[scale=1.3,sibling distance=25mm, level distance=20mm]  \node {$(5b+2c)$}     child {node {$(3b+2c)$}              child {node {$b$}}       child {node {$c$}}}    child {node {$(4b+1c)$}       child {node {}}       child {node {$b$}}}     child {node {$(5b)$}       child {node {}}        child {node {$c$}}     }; \end{tikzpicture}

Warianty te pojawiają się z prawdopodobieństwami równymi odpowiednio:

    \[p_1=\frac{\binom{5}{2}}{\binom{7}{2}}=\frac{10}{21},\quad p_2=\frac{\binom{5}{1}\binom{2}{1}}{\binom{7}{2}}=\frac{10}{21},\quad p_3=\frac{\binom{2}{2}}{\binom{7}{2}}=\frac{1}{21}.\]

Wylosowanie kuli czarnej w drugim losowaniu jest więc równe:

    \[p_1\cdot \frac{2}{5}+p_2\cdot\frac{1}{5}+p_3\cdot\frac{0}{5}=\frac{2}{7}.\]



Zadanie 9. (0-3)
Funkcja f jest określona wzorem f(x)=\dfrac{3x^2-2x}{x^2+2x+8} dla każdej liczby rzeczywistej x. Punkt P=(x_0,3) należy do wykresu funkcji f. Oblicz x_0 oraz wyznacz równanie stycznej do wykresu funkcji f w punkcie P.


Dziedzina: x^2+2x+8\neq 0; warunek ten jest spełniony dla wszystkich x\in\mathbf{R}, gdyż wyróżnik tego trójmianu jest ujemny: \Delta=4-32=-28.

Obliczamy wartość x_0. Aby punkt P należał do wykresu funkcji, musi być f(x_0)=3, stąd

    \[\frac{3x_0^2-2x_0}{x_0^2+2x_0+8}=3\quad\Leftrightarrow\quad -2x_0=6x_0+24,\]

czyli x_0=-3.

Obliczamy pochodną funkcji f:

    \[f'(x)=\frac{(6x-2)(x^2+2x+8)-(3x^2-2x)(2x+2)}{(x^2+2x+8)^2}.\]

Potrzebujemy jedynie wartości f'(-3). Mamy

    \[f'(-3)=\frac{(-20)\cdot 11-33\cdot(-4)}{11^2}=-\frac{8}{11}.\]

Szukana styczna ma zatem równanie

    \[y=-\frac{8}{11}(x+3)+3=-\frac{8}{11}x+\frac{9}{11}.\]



Zadanie 10. (0-4)
Rozwiąż nierówność

    \[\sqrt{x^2+4x+4}<\frac{25}{3}-\sqrt{x^2-6x+9}.\]

Wskazówka: skorzystaj z tego, że \sqrt{a^2}=|a| dla każdej liczby rzeczywistej a.


Ponieważ x^2+4x+4=(x+2)^2 oraz x^2-6x+9=(x-3)^2, to dana nierówność przyjmuje postać

    \[|x+2|+|x-3|<\frac{25}{3}.\]

Prowadzi to do trzech przypadków, które możemy zilustrować na osi liczbowej.

\begin{tikzpicture} \draw[very thick, ->] (-3.5,0)--(3.9,0) node[below] {$x$}; \foreach \x in {-2,0,3}   \draw (\x,0)--(\x,-0.07) node[below] {$\x$}; \draw[thin,blue] (-3.2,0.4)--(-2.2,0.4)--(-2,0.05)--(-1.8,0.4)--(2.8,0.4)--(3,0.05)--(3.2,0.4)--(3.8,0.4); \node[above,red,scale=1.3] at (-2.7,0.3) {$1^\circ$};  \node[above,red,scale=1.3] at (0.5,0.3) {$2^\circ$}; \node[above,red,scale=1.3] at (3.5,0.3) {$3^\circ$};   \end{tikzpicture}

Rozważamy kolejne przypadki. Należy pamiętać o zmianie znaku całego wyrażenia spod wartości bezwzględnej, jeśli w danym przedziale jest ono ujemne.

  • 1^\circ.\quad x\in(-\infty,-2].

    Wtedy x+2\leqslant 0 i podobnie x-3\leqslant 0; zmieniamy więc znak w obu składnikach. Dostajemy zatem nierówność

        \[-(x+2)-(x-3)< \frac{25}{3}\quad\Longleftrightarrow\quad -2x+1< \frac{25}{3}\quad\Longleftrightarrow\quad x> -\frac{11}{3}.\]

    Mamy więc w tym przypadku przedział x\in(-11/3,-2].

  • 2^\circ.\quad x\in(-2,3].
  • Wtedy x+2>0 ale x-3\leqslant 0; zmieniamy znak tylko w drugim składniku. Mamy

        \[(x+2)-(x-3)< \frac{25}{3}\quad\Longleftrightarrow\quad 5< \frac{25}{3},\]

    czyli nierónwość prawdziwą; rozwiązanie stanowi w tym przypadku pełen rozpatrywany zakres (-2,3].

  • 3^\circ.\quad x\in(3,\infty).
  • Wtedy x+2>0 i x-3>0; nie zmieniamy znaków. Mamy

        \[(x+2)+(x-3)< \frac{25}{3}\quad\Longleftrightarrow\quad 2x< \frac{28}{3}\quad\Longleftrightarrow\quad x< \frac{14}{3}.\]

    Ten przypadek prowadzi więc do przedziału (3,14/3).

Na koniec, jako rozwiązanie, bierzemy sumę uzyskanych we wszystkich przypadkach zbiorów, czyli x\in (-3,-2]\cup (-2,3]\cup (3,14/3)=(-3,14/3).



Zadanie 11. (0-4)
Określamy kwadraty K_1,\,K_2,\,K_3,\,\ldots następująco:
  • K_1jest kwadratem o boku długości a
  • K_2 jest kwadratem, którego każdy wierzchołek leży na innym boku kwadratu K_1 i dzieli ten bok w stosunku 1 \colon 3
  • K_3 jest kwadratem, którego każdy wierzchołek leży na innym boku kwadratu K_2 i dzieli ten bok w stosunku 1 \colon 3
i ogólnie, dla każdej liczby naturalnej n\geqslant 2,
  • K_n jest kwadratem, którego każdy wierzchołek leży na innym boku kwadratu K_{n-1} i dzieli ten bok w stosunku 1 \colon 3.
Obwody wszystkich kwadratów określonych powyżej tworzą nieskończony ciąg geometryczny. Na rysunku przedstawiono kwadraty utworzone w sposób opisany powyżej.

\begin{tikzpicture}[scale=1.3] \coordinate (A) at (0,0); \coordinate (B) at (4,0); \coordinate (C) at (4,4); \coordinate (D) at (0,4); \node[below] at (2,0) {$a$}; \node[left] at (0,2) {$a$}; \foreach \x in {1,...,10} {   \draw (A)--(B)--(C)--(D)--cycle;   \coordinate (A1) at ($(A)!0.25!(B)$);   \coordinate (B1) at ($(B)!0.25!(C)$);   \coordinate (C1) at ($(C)!0.25!(D)$);   \coordinate (D1) at ($(D)!0.25!(A)$);     \draw (A1)--(B1)--(C1)--(D1)--cycle;   \coordinate (A) at ($(A1)!0.25!(B1)$);   \coordinate (B) at ($(B1)!0.25!(C1)$);   \coordinate (C) at ($(C1)!0.25!(D1)$);   \coordinate (D) at ($(D1)!0.25!(A1)$); } \draw[thick] (0,0)--(4,0)--(4,4)--(0,4)--cycle; \end{tikzpicture}

Oblicz sumę wszystkich wyrazów tego nieskończonego ciągu.


Niech (r_n) oznacza obwód kwadratu K_n dla n=1,2,\ldots. Mamy oczywiście r_1=4a. Wierzchołki kwadratu K_2 dzielą boki kwadratu K_1 w stosunku 1\colon 3, czyli powstają odcinki długości \dfrac{a}{4} oraz \dfrac{3a}{4}. Zatem bok b kwadratu K_2 spełnia zależność:

    \[b^2=\left(\frac{a}{4}\right)^2+\left(\frac{3a}{4}\right)^2=\frac{a^2}{16}+\frac{9a^2}{16}=\frac{10}{16}a^2,\]

a stąd b=\dfrac{\sqrt{10}}{4}a i r_2=4b=a\sqrt{10}. To już oznacza, że ciąg (r_n), jako ciąg geometryczny, ma iloraz równy q=\dfrac{\sqrt{10}}{4}. Suma obwodów wszystkich opisanych w zadaniu kwadratów wynosi więc

    \[S=\frac{r_1}{1-q}=\frac{4a}{1-\frac{\sqrt{10}}{4}}=\frac{16}{4-\sqrt{10}}a=\frac{8(4+\sqrt{10})}{3}a.\]



Zadanie 12. (0-4)
Rozwiąż równanie 3\sin^2x - \sin^2(2x) = 0 w przedziale [\pi,2\pi].


Wykorzystamy równość \sin 2x=2\sin x\cos x. Wówczas dane równanie przyjmuje postać

    \[3\sin^2x-4\sin^2x\cos^2x=0\quad\Leftrightarrow\quad \sin^2x(3-4\cos^2x)=0.\]

Wynika stąd, że \sin^2 x=0 lub \cos^2x=\dfrac{3}{4}, zatem

    \[\sin x=0\quad\text{lub}\quad \cos x=-\frac{\sqrt{3}}{2}\quad\text{lub}\quad\cos x=\frac{\sqrt{3}}{2}.\]

W przedziale [\pi,\,2\pi] dostajemy stąd x=0 lub x=\pi lub x=\dfrac{7\pi}{6} lub x=\dfrac{11\pi}{6}.



Zadanie 13. (0-4)
Czworokąt ABCD, w którym |BC|=4 i |CD|=5, jest opisany na okręgu. Przekątna AC tego czworokąta tworzy z bokiem BC kąt o mierze 60^\circ, natomiast z bokiem AB – kąt ostry, którego sinus jest równy \dfrac{1}{4}. Oblicz obwód czworokąta ABCD.


Przyjmijmy oznaczenia jak na rysunku, w szczególności \sin\alpha=\dfrac{1}{4}.

\begin{tikzpicture}[scale=0.65] \coordinate (A) at (-7.32,4.94); \coordinate (B) at (6.46,6.56); \coordinate (C) at (7.97,2.85); \coordinate (D) at (5.97,-1.73); \coordinate (O) at (4.3,2.9);  \foreach \x/\w in {A/left,B/above,C/right,D/below}   \draw[fill=black] (\x) circle (1pt) node[\w] {$\x$};  \draw[blue] (A)--(C); \draw[thin] (B)--(C) node[midway,right] {$4$}; \draw[thin] (D)--(C) node[midway,right] {$5$}; \draw[thin] (B)--(A) node[midway,above] {$a$}; \draw[thin] (D)--(A) node[midway,below] {$b$};  \draw[thin,shift={(A)}] (352:3) arc (352:366.5:3) node at (360:2.3) {$\alpha$}; \draw[thin,shift={(C)}] (472:1.4) arc (472:532:1.4) node at (501:0.9) {$60^\circ$};  \draw[thick] (A)--(B)--(C)--(D)--cycle; \draw[red] (O) circle (3.387cm); \end{tikzpicture}

Z warunki opisywalności czworokąta na okręgu mamy a+5=b+4, czyli b=a+1. Z twierdzenia sinusów dla trójkąta ABC możemy napisać

    \[\frac{a}{\sin 60^\circ}=\frac{4}{\sin\alpha}\quad\Rightarrow\quad a=\frac{4}{1/4}\cdot\frac{\sqrt{3}}{2}=8\sqrt{3}.\]

Wówczas b=a+1=8\sqrt{3}+1 i obwód całego czworokąta wynosi a+b+9=16\sqrt{3}+10.



Zadanie 14. (0-4)
Dany jest sześcian ABCDEFGH o krawędzi długości 6. Punkt S jest punktem przecięcia przekątnych AH i DE ściany bocznej ADHE (zobacz rysunek).

\begin{tikzpicture}[scale=0.9] \coordinate (A) at (0,0); \coordinate (B) at (5,0); \coordinate (C) at (7,2); \coordinate (D) at (2,2); \coordinate (E) at (0,5); \coordinate (F) at (5,5); \coordinate (G) at (7,7); \coordinate (H) at (2,7); \coordinate (S) at (1,3.5); \draw[thick] (A)--(B)--(C)--(G)--(H)--(E)--cycle (F)--(G) (E)--(F)--(B); \draw[thick,dashed] (A)--(D)--(H) (D)--(C); \draw[dotted,thick,black!75] (A)--(H) (E)--(D); \node[below] at (2.5,0) {$6$};  \foreach \x/\r in {A/{below left},B/{below right},C/right,D/{below right},E/left,F/right,G/{above right},H/above,S/left}   \draw[fill=black] (\x) circle (1pt) node[\r] {$\x$}; \end{tikzpicture}

Oblicz wysokość trójkąta SBH poprowadzoną z punktu S na bok BH tego trójkąta.

Oznaczmy szukaną wysokość przez d. Pole P trójkąta BHS stanowi połowę pola trójkąta ABH, bo S jest środkiem boku AH. Z drugiej strony trójkąt ABH jest połową prostokąta ABGH. Stąd

    \[P=\frac{1}{4}|AB|\cdot|BG|=\frac{1}{4}\cdot 6\cdot 6\sqrt{2}=9\sqrt{2}.\]

\begin{tikzpicture}[scale=0.9] \coordinate (A) at (0,0); \coordinate (B) at (5,0); \coordinate (C) at (7,2); \coordinate (D) at (2,2); \coordinate (E) at (0,5); \coordinate (F) at (5,5); \coordinate (G) at (7,7); \coordinate (H) at (2,7); \coordinate (S) at (1,3.5);  \coordinate (K) at ($(B)!(S)!(H)$);  \draw[fill=yellow!30] (A)--(B)--(G)--(H)--cycle; \draw[fill=blue!30] (S)--(B)--(H)--cycle;  \draw[red,thick] (S)--(K) node[below right,midway] {$d$};  \draw[thick] (A)--(B)--(C)--(G)--(H)--(E)--cycle (F)--(G) (E)--(F)--(B); \draw[thick,dashed] (A)--(D)--(H) (D)--(C); \draw[dotted,thick,black!75] (A)--(H) (E)--(D); \node[below] at (2.5,0) {$6$};  \foreach \x/\r in {A/{below left},B/{below right},C/right,D/{below right},E/left,F/right,G/{above right},H/above,S/left}   \draw[fill=black] (\x) circle (1pt) node[\r] {$\x$}; \end{tikzpicture}

Ponieważ przekątna BH danego sześcianu ma długość |BH|=6\cdot\sqrt{3}, to szukana wysokość ma długość

    \[d=\frac{2P}{|BH|}=\frac{18\sqrt{2}}{6\sqrt{3}}=3\sqrt{\frac{2}{3}}=\sqrt{6}.\]

 

Zadanie 15. (0-5)
Wyznacz wszystkie wartości parametru m\neq 2, dla których równanie

    \[x^2+4x-\frac{m-3}{m-2}=0\]

ma dwa różne rozwiązania rzeczywiste x_1, x_2 spełniające warunek x_1^3+x_2^3>-28.

Aby spełnione były warunki nałożone przez treść zadania na daną funkcję kwadratową, muszą zachodzić zależności:

    \[\Delta >0\quad\wedge\quad x_1^3+x_2^3>-28.\]

Ostatnią nierówność możemy zapisać jako

    \[(x_1+x_2)(x_1^2-x_1x_2+x_2^2)=(x_1+x_2)\big[(x_1+x_2)^2-3x_1x_2\big]>-28,\]

i, dzięki wzorom Viete’a, mamy

    \[-4\left((-4)^2-3\cdot\left(-\frac{m-3}{m-2}\right)\right)>-28.\]

Stąd 16+\dfrac{3(m-3)}{m-2}<7 i dalej \dfrac{9(m-2)+3(m-3)}{m-2}<0. Dzieląc obustronnie przez 3 dostaniemy

    \[\frac{4m-9}{m-3}<0\quad\Leftrightarrow\quad m\in\left(\frac{9}{4},\,3\right).\]

Podobnie warunek z wyróżnikiem:

    \[\Delta=16+4\cdot\frac{m-3}{m-2}=4\cdot\frac{5m-11}{m-2}>0\quad\Leftrightarrow\quad m\in\mathbf{R}\setminus\left[2,\,\frac{11}{5}\right].\]

Ostatecznie, biorąc część wspólną obu otrzymanych zbiorów, uzyskujemy końcową odpowiedź: m\in\left(\dfrac{9}{4},\,3\right)



Zadanie 16. (0-7)
Rozważamy trójkąty ABC, w których A=(0,0), B=(m,0), gdzie m\in(4,+\infty), a wierzchołek C leży na prostej o równaniu y=-2x. Na boku BC tego trójkąta leży punkt D=(3,2).

a) Wykaż, że dla m\in(4,+\infty) pole P trójkąta ABC, jako funkcja zmiennej m, wyraża się wzorem

    \[P(m)=\frac{m^2}{m-4}.\]

b) Oblicz tę wartość m, dla której funkcja P osiąga wartość najmniejszą. Wyznacz równanie prostej BC, przy której funkcja B osiąga tę najmniejszą wartość.


Wyznaczymy najpierw współrzędne punktu C=(x_C,y_C) w zależności od m. Równanie prostej przechodzącej przez punkty B i D ma postać

    \[y=\frac{0-2}{m-3}(x-m)\quad\Leftrightarrow\quad y=\frac{-2}{m-3}x+\frac{2m}{m-3}.\]

Prosta ta przecina prostą y=-2x w punkcie C, którego współrzędne (x_C,y_C) spełniają układ równań

    \[\left\{\begin{array}{l}y=\frac{-2}{m-3}x+\frac{2m}{m-3}\\ y=-2x\end{array}\right..\]

\begin{tikzpicture}[scale=1] \coordinate (A) at (0,0); \coordinate (B) at (6,0); \coordinate (D) at (3,2); \coordinate (C) at (-3,6); \draw[fill=yellow!40] (A)--(B)--(C)--cycle;  \draw[black!70] (-6.4,-1.2) grid (6.3,8.3); \draw[very thick,->] (-6.4,0)--(6.3,0) node[below] {$x$}; \draw[very thick,->] (0,-1.2)--(0,8.3) node[left] {$y$}; \draw[thick,blue,dashed] (-4.3,8.6)--(1,-2) node[below,sloped,midway] {$y=-2x$}; \draw[very thick,brown,dashed] (4,-1.2)--(4,8.3);  \foreach \x in {A,B,D,C}    \draw[fill=black] (\x) circle (1.7pt) node[below left] {$\x$};    \node[below left] at (4,0) {$4$}; \node[above] at (B) {$(m,0)$}; \node[above] at (D) {$(3,2)$}; \end{tikzpicture}

To oznacza, że \displaystyle -\frac{2x}{m-3}+\frac{2m}{m-3}=-2x, a stąd \left(\dfrac{2}{m-3}-2\right)x=\dfrac{2m}{m-3} i

    \[x=\frac{m}{4-m}\quad\text{dla}\quad m>4.\]

Wtedy oczywiście y=-2x=\dfrac{2m}{m-4}.

a) Pole P(m) trójkąta ABC jest równe

    \[P=\dfrac{1}{2}\cdot |AB|\cdot y_C=\frac{1}{2}\cdot m\cdot \frac{2m}{m-4}=\frac{m^2}{m-4}\quad\text{dla}\quad m>4.\]

b) Znajdziemy minimum funkcji P(m) dla m>4. Obliczamy pochodną:

    \[P'(m)=\frac{2m(m-4)-m^2}{(m-4)^2}=\frac{m(m-8)}{(m-4)^2}.\]

Widać, że dla m>4 jedynym miejscem zerowym pochodnej jest m=8. W punkcie tym pochodna zmienia swój znak z ujemnego na dodatni (bo tak zachowuje się funkcja kwadratowa m\mapsto m(m-8)), wobec tego funkcja P(m) osiąga dla m=8 swoje minimum. Wtedy prosta BC ma równanie:

    \[y=-\frac{2}{5}x+\frac{16}{5}.\]



Dodaj komentarz

Twój adres e-mail nie zostanie opublikowany.